I have a horizontally supported cylindrical tube that is fixed at both ends. I modeled half of it to run the simulation on in Abaqus. The only load that it has is its weight. What I get confused at with cylindrical geometry is applying the load. How should I apply the gravitational load and at...
Hello, PF! Recently, while reading chapter 10 (microscopic energy balances) of the second edition of BSL, I found a minor discrepancy which is confusing me, especially when considering the mathematical analogies of heat and mass transfer.
In section 10.1, the authors introduce Newton's law of...
I want to know how this integral will equal zero?
I know that Ψ will fall to zero as x goes to infinity
and i know that Ψ must fall to zero very quickly , Ψ must fall to zero faster than 1/√|x|
all of this will help evaluating this integral
i tried to solve it as follows
The first term...
How to find value of integration constant?I know with the help of boundary conditions,but How boundary conditions help in finding integration constant?
I want to verify whether I am correct .As the title suggests It is about point where potential is zero (point other than infinity)
Note:x is distance of neutral point from charges
Firstly,I would like to ask
Is coordinate system necessarily be there
1) If there are two positive charges for the...
Homework Statement
Under what conditions is \left\langle{{\mathbf{x} \cdot \mathbf{p}}}\right\rangle a constant.
A proof of the quantum virial theorem starts with the computation of the commutator of \left[{\mathbf{x} \cdot \mathbf{p}},{H}\right] . Using that one can show for Heisenberg...
Hello,
I have a question regarding the solution to a second order 'mass-spring-damper' system. Over the years, I have gotten familiar with the idea of system damping in the sense of under damped, over damped, and critically damped systems.
However, I've began looking closer at the solution to...
Suppose y is a positive vector. Let p and x be two positive matrices with N rows, where ##p_j## and ##x_j## denotes the j:th row in these matrices, so that j = 1,…,N.
Does the following hold:
\inf_{k=1,...,N} [\sup_{l=1,...,N} [p_k(y-x_k)]] = \inf_{k=1,...,N} [p_k(y-x_k)]
where ##p_k(y-x_k)##...
Mod note: Fixed the broken LaTeX in the following, and edited the difference quotient definition of f'(z).
The following is from a mathematical introduction to a physics book:
"The real part u and the imaginary part v of w=u+i v are functions of the two variables x and y. Nevertheless, two...
Hi,
We normally use a simple symmetry argument to show that the probability of each outcome of a throw of a fair, cube-shaped die is 1/6. However, is it possible to actually model the physics of the throw and show that the probabilities are 1/6?
Since this is classical physics, the outcome can...
Hi, just want to confirm that with the eigenfunction boundary condition $ p(x) v^*(x)u'(x)|_{x=a} = 0 $, the order of (solutions) v, u doesn't matter? I ask because a problem like this had one solution = a constant, so making that the u solution makes $ p(x) v^*(x)u'(x) = 0 $ no matter the...
I am reading Jackson Electrodynamics (section 1.10 in 3rd edition) and he is discussing the Poisson eqn $$\nabla^2 \Phi = -\rho / \epsilon_0$$ defined on some finite volume V, the solution using Greens theorem is
$$\Phi (x) = \frac{1}{4 \pi \epsilon_0} \int_V G(x,x') \rho(x')d^3x' +\frac{1}{4...
I've forgotten a lot of field theory so I've been rereading it in a couple of electric field theory textbooks. What seems like a simple problem falls between the cracks. I hope some readers can help - it will be appreciated.
My application seems simple (solution will require numerical FEA but...
Hi Everyone,
I had a quick question. If you have an IVP ODE and you solve for the general solution first and you had fractions in it, could you multiply by a number to make it "easier" (whole number, rather than involving fractions) without violating the initial conditions?
Thanks
I am working with a fixed fixed bar with a distributed axial load to the right as w(x)=CX/L. I am having a hard time determining the force boundary conditions. I know that U(0)=0 and U(L)=0. However, I need to come up with something in regards to U'(Value). Any help would be appreciated.
Hey! :o
I want to prove the following lemma:
Let $n>1$. We suppose that the integers $x$ and $y$ satisfy the relations $x \mid_n 1$ and $y \mid_n 1$. Then $y=x^2$ if and only if the following conditions hold:
$2nx+1 \mid_n 4n^2y-1$
$2nx-1 \mid_n 4n^2y-1$
$ny-kx \mid_n nx-k$, for each...
Hi,
I have two coupled differential equations
d^2 phi(z)/dz^2=lambda*phi(z)*(phi(z)^2+psi(z)^2-sigma^2)
d^2 psi(z)/dz^2=lambda*psi(z)*(phi(z)^2+psi(z)^2-sigma^2+epsilon/lambda)
where lambda, epsilon and sigma are arbitrary constants. The equation subject to the bellow boundary conditions...
Consider the heat equation
dT/dt - aΔT + v⋅∇T = S
where S is a source term dependent of the radiation intensity I and the temperature T. The fluid velocity v is prescribed.
We also consider the radiative transfer equation describing the radiative intensity I(x,ω,t) where ω is the ray direction...
1) Are there any ways to control the impulse in an collision in a lab condition? I.e. if I set an object connected to a spring and try to hit a ball using the object, are there any ways to control the impulse of the collision?
2) How to detect the change from slipping to pure rolling...
I'm currently a mechanical engineering student and many courses that I take involve topics that have multiple conditions. For instance if event "A" happens then apply Table "a" or Equation "aa". It becomes difficult to solve problems when topics have multiple conditions especially when similar...
Shouldn't the ΔG° partial pressure of the components be based on the K? Where ΔG°=-RTlnK such that K is based on the partial pressures of the gas involved?
If we were to set the partial pressure to be 1bar each then every reaction having the same stoichiometric proportion of reactants and...
I am trying to calculate the ##\beta## functions of the massless pseudoscalar Yukawa theory, following Peskin & Schroeder, chapter 12.2. The Lagrangian is
##{L}=\frac{1}{2}(\partial_\mu \phi)^2-\frac{\lambda}{4!}\phi^4+\bar{\psi}(i\gamma^\mu \partial_\mu)\psi-ig\bar{\psi}\gamma^5\psi\phi.##...
The FRW cosmology is a solution of the FRW that can be foliated into 3D isotropic and homogeneous slices.
This foliation is implemented mathematically first by the use of a not generally covariant coordinate condition https://en.wikipedia.org/wiki/Coordinate_conditions#Synchronous_coordinates ...
ΔG=ΔG°+RTlnQ are for reactions that are not under standard conditions. What does it mean under non-standard conditions for the ΔG?
And what is the Q in this case? Is it Qp where it is based on the partial pressure of the gas or Qc which is based on the concentration?
Thanks
Hello,
I am working on a hydraulic cylinder application that will lift some weight that is not aligned with the axis of the cylinder. Take a look at the picture and imagine that instead of ropes there is a hydraulic cylinder fixed to the beam doing the lifting. The load creates moment since it...
Hello!
I'm currently making my way through the book "Quantum Field Theory of Point Particles and Strings" and on page 13 they talk are talking about quantization of the classical versions momentum and position. The first part to quantizing these is turning them into operators. The books goes on...
Homework Statement
Write down the geodesic equation. For ##x^0 = c\tau## and ##x^i = constant##, find the condition on the christoffel symbols ##\Gamma^\mu~_{\alpha \beta}##. Show these conditions always work when the metric is of the form ##ds^2 = -c^2dt^2 +g_{ij}dx^idx^j##.Homework...
I am trying to set up the mass matrix for a 1D system which I want to solve using finite elements. So the mass matrix is defined as
M = \int{NN^T}dL,
where N is the finite element linear basis functions. I use hat functions.
Say I have 10 elements, corresponding to 11 nodes running from -5...
I am not sure if this should be posted here. If not I hope you accept my apologizes and the admin move on the post as soon as possible.
I am studying manifold with boundaries and boundary conditions in a quantum field theory approach.
Could you recommend me books or papers about that?
I lack...
I read about the wheeler experiment been done in Australia. But one thing I don't understand. If the particle (but let's use a photon) goes through the two slits (or one) and the screen is removed after it already went through, how is it measured?
Does it stop after the measurement? My...
Hi! When we model bloch-waves in a solid we assume that there exist some kind of periodic boundary conditions such that the wave function is periodic. In 1D, ##\psi(x)## repeats itself for every ##L##, ##\psi(x) = \psi(x+L)##, such as here:
OK, fine, we get pretty wave solutions if we assume...
Question about conditions for conservative field
In common textbooks' discussions about conservative vector field. There is always two assumptions about the region concerned, namely the region is simply connected and open.
Usually in textbooks there is not much explanations on why these...
we have that Ht1 (x,y,z) - Ht2 (x,y,z) = Js and for the special case Ht1 (x,y,z) - Ht2 (x,y,z) = 0 where there is no surface current. At a boundary with Js =0, which for simplicity let's asume is at at x = a, then knowing that Ht1 and Ht2 are the magnetic fields to the left and right of the...
Let M = {x1, x2, x3, ...} U {p} be a perfect metric space.
Let f be continuous, taking M to M with f(xn) = xn+1 and f(p) = p.
I would like to know if this dynamical system is necessarily sensitive to initial conditions.
Homework Statement
clear all;
close all;
totalanswers=zeros(100,4);
k= ;
flag =0;
for i =0:25
a=i;
if flag ==1
break;
end
and so forth for a,b,c,d, etc for a finite set of values all have the break values too.
x=[ a,b,c,d];
does some math calculationhere is a...
What are the conditions necesairy to collapse a wave function of a electron, photon or an atom (alpha particle mostly?).
Wavelength? Asking because I cannot understand why in this video the wave function does not collapse in this box.
Seems like in the outside there should be massive...
Consider the radial differential equation
##\bigg( - \frac{d^2}{dr^2} + \frac{(l+\frac{d-3}{2})(l+\frac{d-1}{2})}{r^2} + V(r) + m^2 \bigg) \phi_l (r) = \lambda\ \phi_l (r)##,
which I've obtained by solving the Schrodinger equation in ##d## dimensions using the method of separation of...
I have a question I'm a little embarrassed to be asking: what is meant in condensed matter when someone describes a system with "open boundary conditions," say in one-dimension for simplicity? I am comfortable with the statement of fixed (Dirichlet) or free (von Neumann) boundary conditions, as...
Hi all!
I am trying to solve a system of partial differential equations in Matlab, with both derivatives in time and space domains. I am using the pdepe function for that.
The system is, to be simple, a sort of solar thermal panel, made of three layers: an absorber plate, a fluid layer of...
I know this is the Physics Forums and not the Biology Forums, but I feel that this question is more on the physics side of the spectrum than the biology side.
What potential effects could the existence of a hypothetical, unusually warped spacetime, where there were naturally-occurring curves...
I have created such vibrating feeder model on SW but on ANSYS I'm only analyzing the frame.
1)How do I set up boundary conditions, I think I need elastic supports?
2)In modal analysis I have elastic, fixed supports, remote displacement - which should I use and in what directions?
3)Do I perform...
Homework Statement
This is not homework but I am trying to solve a problem to see to general a set of numbers for the grassmanian equations to use in something else.[/B]
I have five variables a,b,c,d,e. a-b,a-c,a-d,a-e,b-a,b-c,...etc so that none of them are the same so it generates all...
I'm reading griffiths electrodynamics and I am confused about a concept. Mainly because I might be interpreting it in different ways. Why does the equation contain an E with a negative in front? Namely, E_below. Isn't the Electric field pointing away from the surface with the surface charge...
Edit: Sorry about the vague title, it was intended to be complex beam system boundary conditions but somehow it turned out like this.
Hello,
I am trying to learn complex beam system designs and I sometimes struggle to assign boundary conditions. For example I am trying to design the lifting...
In the derivation of the boundary conditions we apply the integral form of maxwell's equations, but once we take a very small volume we find that some terms disappear like the displacement current as well as the time derivative of the magnetic field. Why do these terms disappear? For reference...
Does anyone here have a copy of Griffith's E&M?
On page 128, condition III V=V_0 (y) when x = 0.
Do you know why then value V_0(y) does not appear in in equation 3.28, V(x,y) = Ce^(-ky)sin(ky)?
The author does not explain this.
I stumbled upon this article: http://www.comsol.com/blogs/exploiting-symmetry-simplify-magnetic-field-modeling/
Since the article does not contain any mathematical formulations, I was wondering how the boundary conditions can be expressed in terms of magnetic vector potential.
From what I...
Hello,
I need to replicate the conditions of a clutch on a pin on disc tribometer to measure the coefficient of friction. However the tribometer I have access to can't spin faster than 1000 RPM. The clutch will reach speeds from 3000 RPM to 9000 RPM. I calculated the normal force required on...